Evaluating the sum of the series $sumlimits_{n=1}^{infty} frac{1}{n^{3} binom{2n}{n}} $












38












$begingroup$


Wolfram MathWorld states that $$ sum_{n=1}^{infty} frac{1}{n^{3} binom{2n}{n}} = frac{ pi sqrt{3}}{18} Bigg[ psi_{1} left(frac{1}{3} right) - psi_{1} left(frac{2}{3} right) Bigg]- frac{4}{3} zeta(3) , , $$



where $psi_{1}(x)$ is the trigamma function.



I can't seem to get the answer in that form.



Using the Taylor expansion $ displaystyle arcsin^{2}(x) = frac{1}{2} sum_{n=1}^{infty} frac{1}{n^{2} binom{2n}{n}} (2x)^{2n}$, I get



$$ sum_{n=1}^{infty} frac{1}{n^{3} binom{2n}{n}} = 4 int_{0}^{frac{1}{2}} frac{arcsin^{2}(x)}{x} , dx. $$



Then I integrating by parts, I get



$$ begin{align} &4 int_{0}^{frac{1}{2}} frac{arcsin^{2}(x)}{x} , dx \ &= frac{pi^{2}}{9} ln left(frac{1}{2} right) - 8 int_{0}^{frac{1}{2}} frac{arcsin (x) ln (x)}{sqrt{1-x^{2}}} , dx \ &= - frac{pi^{2}}{9} ln 2 - 8 int_{0}^{frac{pi}{6}} u ln (sin u ) , du \ &= - 8 ln 2 int_{0}^{frac{pi}{6}} u , du - 8int_{0}^{frac{pi}{6}} u ln (sin u ) , du \ &= - 8 int_{0}^{frac{pi}{6}} u ln ( 2 sin u ) , du \ &= -8 text{Re} int_{0}^{frac{pi}{6}} u ln (1-e^{2iu}) , du \ &= 8 text{Re} int_{0}^{frac{pi}{6}} u sum_{n=1}^{infty} frac{e^{2in u}}{n} , du \ &= 8 sum_{n=1}^{infty} frac{1}{n} int_{0}^{frac{pi}{6}} u cos (2nu) , du \ &= frac{2 pi}{3} sum_{n=1}^{infty} frac{sin (frac{n pi}{3})}{n^{2}} + 2 sum_{n=1}^{infty} frac{cos (frac{n pi}{3})}{n^{3}} - 2 zeta(3) \ &= frac{2 pi}{3} Bigg( frac{sqrt{3}}{2} sum_{n=0}^{infty} frac{1}{(6n+1)^{2}} + frac{sqrt{3}}{2} sum_{n=0}^{infty} frac{1}{(6n+2)^{2}} - frac{sqrt{3}}{2} sum_{n=0}^{infty} frac{1}{(6n+4)^{2}} - frac{sqrt{3}}{2} sum_{n=0}^{infty} frac{1}{(6n+5)^{2}} Bigg) \ &+ 2 Bigg( frac{1}{2} sum_{n=0}^{infty} frac{1}{(6n+1)^{3}} - frac{1}{2} sum_{n=0}^{infty} frac{1}{(6n+2)^{3}} - sum_{n=0}^{infty} frac{1}{(6n+3)^{3}} - frac{1}{2} sum_{n=0}^{infty} frac{1}{(6n+4)^{3}} \ &+ frac{1}{2} sum_{n=0}^{infty} frac{1}{(6n+5)^{3}} + sum_{n=1}^{infty} frac{1}{(6n)^{3}} Bigg) - 2 zeta(3) \ &= frac{pi sqrt{3}}{108} Bigg( psi_{1} left(frac{1}{6} right) + psi_{1} left(frac{1}{3} right) - psi_{1} left(frac{2}{3} right) - psi_{1}left(frac{5}{6} right) Bigg) + frac{1}{432} Bigg( - psi_{2} left(frac{1}{6} right) + psi_{2} left(frac{1}{3} right) \ &-28 zeta(3) + psi_{2} left(frac{2}{3} right) - psi_{2} left(frac{5}{6} right) + 4 zeta(3)Bigg) - 2 zeta (3) .end{align}$$



From here I've been going in circles trying to get the answer in the form given above.





EDIT:



Using the duplication formula for the trigamma function (i.e., $ displaystyle 4 psi_{1}(2x) = psi_{1}(x) + psi_{1} left(x + frac{1}{2} right) $),



$$ begin{align} &psi_{1} left(frac{1}{6} right) + psi_{1} left(frac{1}{3} right) - psi_{1} left(frac{2}{3} right) - psi_{1}left(frac{5}{6} right) \ &= 4 psi_{1} left(frac{1}{3} right) -psi_{1} left(frac{2}{3} right) + psi_{1} left(frac{1}{3} right) -psi_{1} left(frac{2}{3} right) - 4 psi_{1} left(frac{2}{3} right) + psi_{1} left(frac{1}{3} right) \ &= 6 psi_{1} left(frac{1}{3} right) - 6 psi_{1} left(frac{2}{3} right). end{align}$$



Therefore,



$$ begin{align} sum_{n=1}^{infty} frac{1}{n^{3} binom{2n}{n}} &= frac{sqrt{3} pi}{18} Bigg( psi_{1} left(frac{1}{3} right) - psi_{1} left(frac{2}{3} right) Bigg) +
frac{1}{432} Bigg( - psi_{2} left(frac{1}{6} right) + psi_{2} left(frac{1}{3} right) \ &+ psi_{2} left(frac{2}{3} right) - psi_{2} left(frac{5}{6} right) - 2 zeta(3)Bigg) - 2 zeta (3). end{align}$$



So it comes down to somehow showing that $$-psi_{2} left(frac{1}{6} right) + psi_{2} left(frac{1}{3} right) + psi_{2} left(frac{2}{3} right) - psi_{2} left(frac{5}{6} right) = 312 zeta(3) .$$



SECOND EDIT:



Using the duplication formula for $psi_{2}(x)$, we get



$$ begin{align} &-psi_{2} left(frac{1}{6} right) + psi_{2} left(frac{1}{3} right) + psi_{2} left(frac{2}{3} right) - psi_{2} left(frac{5}{6} right) \ &= -8 psi_{2} left(frac{1}{3} right) +psi_{2} left(frac{2}{3} right) + psi_{2} left(frac{1}{3} right) +psi_{2} left(frac{2}{3} right) - 8 psi_{2} left(frac{2}{3} right) + psi_{2} left(frac{1}{3} right) \ &= -6 psi_{2} left(frac{1}{3} right) - 6 psi_{2} left(frac{2}{3} right) . end{align}$$



And using the more general multiplication formula, we get



$$ psi_{2} left(frac{1}{3} right) + psi_{2} left( frac{2}{3} right) + psi_{2}(1) = 27 psi_{2} (1) .$$



Therefore,



$$ -6 psi_{2} left(frac{1}{3} right) - 6 psi_{2} left(frac{2}{3} right) = -156 psi_{2} (1) = 312 zeta(3) .$$










share|cite|improve this question











$endgroup$












  • $begingroup$
    Isn't this the sum that came up in Apery's proof of the irrationality of $zeta(3)$? If so, it may be in Alf van der Poorten's paper, "A Proof that Euler Missed". Also, I think the sum may be done in Comtet's book on Combinatorics. Also, look at the "Related" questions running down the side of this page.
    $endgroup$
    – Gerry Myerson
    Sep 21 '13 at 1:15












  • $begingroup$
    That sum is the alternating version of this one.
    $endgroup$
    – Random Variable
    Sep 21 '13 at 1:31










  • $begingroup$
    And the techniques that apply to the alternating version don't work for the one you ask about?
    $endgroup$
    – Gerry Myerson
    Sep 21 '13 at 1:41










  • $begingroup$
    They work to a point. But the integrand of the other one is $ displaystyle frac{text{arcsinh}^{2}(x)}{x}$.
    $endgroup$
    – Random Variable
    Sep 21 '13 at 1:56












  • $begingroup$
    use Clausen function
    $endgroup$
    – alexjo
    Nov 13 '13 at 21:08
















38












$begingroup$


Wolfram MathWorld states that $$ sum_{n=1}^{infty} frac{1}{n^{3} binom{2n}{n}} = frac{ pi sqrt{3}}{18} Bigg[ psi_{1} left(frac{1}{3} right) - psi_{1} left(frac{2}{3} right) Bigg]- frac{4}{3} zeta(3) , , $$



where $psi_{1}(x)$ is the trigamma function.



I can't seem to get the answer in that form.



Using the Taylor expansion $ displaystyle arcsin^{2}(x) = frac{1}{2} sum_{n=1}^{infty} frac{1}{n^{2} binom{2n}{n}} (2x)^{2n}$, I get



$$ sum_{n=1}^{infty} frac{1}{n^{3} binom{2n}{n}} = 4 int_{0}^{frac{1}{2}} frac{arcsin^{2}(x)}{x} , dx. $$



Then I integrating by parts, I get



$$ begin{align} &4 int_{0}^{frac{1}{2}} frac{arcsin^{2}(x)}{x} , dx \ &= frac{pi^{2}}{9} ln left(frac{1}{2} right) - 8 int_{0}^{frac{1}{2}} frac{arcsin (x) ln (x)}{sqrt{1-x^{2}}} , dx \ &= - frac{pi^{2}}{9} ln 2 - 8 int_{0}^{frac{pi}{6}} u ln (sin u ) , du \ &= - 8 ln 2 int_{0}^{frac{pi}{6}} u , du - 8int_{0}^{frac{pi}{6}} u ln (sin u ) , du \ &= - 8 int_{0}^{frac{pi}{6}} u ln ( 2 sin u ) , du \ &= -8 text{Re} int_{0}^{frac{pi}{6}} u ln (1-e^{2iu}) , du \ &= 8 text{Re} int_{0}^{frac{pi}{6}} u sum_{n=1}^{infty} frac{e^{2in u}}{n} , du \ &= 8 sum_{n=1}^{infty} frac{1}{n} int_{0}^{frac{pi}{6}} u cos (2nu) , du \ &= frac{2 pi}{3} sum_{n=1}^{infty} frac{sin (frac{n pi}{3})}{n^{2}} + 2 sum_{n=1}^{infty} frac{cos (frac{n pi}{3})}{n^{3}} - 2 zeta(3) \ &= frac{2 pi}{3} Bigg( frac{sqrt{3}}{2} sum_{n=0}^{infty} frac{1}{(6n+1)^{2}} + frac{sqrt{3}}{2} sum_{n=0}^{infty} frac{1}{(6n+2)^{2}} - frac{sqrt{3}}{2} sum_{n=0}^{infty} frac{1}{(6n+4)^{2}} - frac{sqrt{3}}{2} sum_{n=0}^{infty} frac{1}{(6n+5)^{2}} Bigg) \ &+ 2 Bigg( frac{1}{2} sum_{n=0}^{infty} frac{1}{(6n+1)^{3}} - frac{1}{2} sum_{n=0}^{infty} frac{1}{(6n+2)^{3}} - sum_{n=0}^{infty} frac{1}{(6n+3)^{3}} - frac{1}{2} sum_{n=0}^{infty} frac{1}{(6n+4)^{3}} \ &+ frac{1}{2} sum_{n=0}^{infty} frac{1}{(6n+5)^{3}} + sum_{n=1}^{infty} frac{1}{(6n)^{3}} Bigg) - 2 zeta(3) \ &= frac{pi sqrt{3}}{108} Bigg( psi_{1} left(frac{1}{6} right) + psi_{1} left(frac{1}{3} right) - psi_{1} left(frac{2}{3} right) - psi_{1}left(frac{5}{6} right) Bigg) + frac{1}{432} Bigg( - psi_{2} left(frac{1}{6} right) + psi_{2} left(frac{1}{3} right) \ &-28 zeta(3) + psi_{2} left(frac{2}{3} right) - psi_{2} left(frac{5}{6} right) + 4 zeta(3)Bigg) - 2 zeta (3) .end{align}$$



From here I've been going in circles trying to get the answer in the form given above.





EDIT:



Using the duplication formula for the trigamma function (i.e., $ displaystyle 4 psi_{1}(2x) = psi_{1}(x) + psi_{1} left(x + frac{1}{2} right) $),



$$ begin{align} &psi_{1} left(frac{1}{6} right) + psi_{1} left(frac{1}{3} right) - psi_{1} left(frac{2}{3} right) - psi_{1}left(frac{5}{6} right) \ &= 4 psi_{1} left(frac{1}{3} right) -psi_{1} left(frac{2}{3} right) + psi_{1} left(frac{1}{3} right) -psi_{1} left(frac{2}{3} right) - 4 psi_{1} left(frac{2}{3} right) + psi_{1} left(frac{1}{3} right) \ &= 6 psi_{1} left(frac{1}{3} right) - 6 psi_{1} left(frac{2}{3} right). end{align}$$



Therefore,



$$ begin{align} sum_{n=1}^{infty} frac{1}{n^{3} binom{2n}{n}} &= frac{sqrt{3} pi}{18} Bigg( psi_{1} left(frac{1}{3} right) - psi_{1} left(frac{2}{3} right) Bigg) +
frac{1}{432} Bigg( - psi_{2} left(frac{1}{6} right) + psi_{2} left(frac{1}{3} right) \ &+ psi_{2} left(frac{2}{3} right) - psi_{2} left(frac{5}{6} right) - 2 zeta(3)Bigg) - 2 zeta (3). end{align}$$



So it comes down to somehow showing that $$-psi_{2} left(frac{1}{6} right) + psi_{2} left(frac{1}{3} right) + psi_{2} left(frac{2}{3} right) - psi_{2} left(frac{5}{6} right) = 312 zeta(3) .$$



SECOND EDIT:



Using the duplication formula for $psi_{2}(x)$, we get



$$ begin{align} &-psi_{2} left(frac{1}{6} right) + psi_{2} left(frac{1}{3} right) + psi_{2} left(frac{2}{3} right) - psi_{2} left(frac{5}{6} right) \ &= -8 psi_{2} left(frac{1}{3} right) +psi_{2} left(frac{2}{3} right) + psi_{2} left(frac{1}{3} right) +psi_{2} left(frac{2}{3} right) - 8 psi_{2} left(frac{2}{3} right) + psi_{2} left(frac{1}{3} right) \ &= -6 psi_{2} left(frac{1}{3} right) - 6 psi_{2} left(frac{2}{3} right) . end{align}$$



And using the more general multiplication formula, we get



$$ psi_{2} left(frac{1}{3} right) + psi_{2} left( frac{2}{3} right) + psi_{2}(1) = 27 psi_{2} (1) .$$



Therefore,



$$ -6 psi_{2} left(frac{1}{3} right) - 6 psi_{2} left(frac{2}{3} right) = -156 psi_{2} (1) = 312 zeta(3) .$$










share|cite|improve this question











$endgroup$












  • $begingroup$
    Isn't this the sum that came up in Apery's proof of the irrationality of $zeta(3)$? If so, it may be in Alf van der Poorten's paper, "A Proof that Euler Missed". Also, I think the sum may be done in Comtet's book on Combinatorics. Also, look at the "Related" questions running down the side of this page.
    $endgroup$
    – Gerry Myerson
    Sep 21 '13 at 1:15












  • $begingroup$
    That sum is the alternating version of this one.
    $endgroup$
    – Random Variable
    Sep 21 '13 at 1:31










  • $begingroup$
    And the techniques that apply to the alternating version don't work for the one you ask about?
    $endgroup$
    – Gerry Myerson
    Sep 21 '13 at 1:41










  • $begingroup$
    They work to a point. But the integrand of the other one is $ displaystyle frac{text{arcsinh}^{2}(x)}{x}$.
    $endgroup$
    – Random Variable
    Sep 21 '13 at 1:56












  • $begingroup$
    use Clausen function
    $endgroup$
    – alexjo
    Nov 13 '13 at 21:08














38












38








38


10



$begingroup$


Wolfram MathWorld states that $$ sum_{n=1}^{infty} frac{1}{n^{3} binom{2n}{n}} = frac{ pi sqrt{3}}{18} Bigg[ psi_{1} left(frac{1}{3} right) - psi_{1} left(frac{2}{3} right) Bigg]- frac{4}{3} zeta(3) , , $$



where $psi_{1}(x)$ is the trigamma function.



I can't seem to get the answer in that form.



Using the Taylor expansion $ displaystyle arcsin^{2}(x) = frac{1}{2} sum_{n=1}^{infty} frac{1}{n^{2} binom{2n}{n}} (2x)^{2n}$, I get



$$ sum_{n=1}^{infty} frac{1}{n^{3} binom{2n}{n}} = 4 int_{0}^{frac{1}{2}} frac{arcsin^{2}(x)}{x} , dx. $$



Then I integrating by parts, I get



$$ begin{align} &4 int_{0}^{frac{1}{2}} frac{arcsin^{2}(x)}{x} , dx \ &= frac{pi^{2}}{9} ln left(frac{1}{2} right) - 8 int_{0}^{frac{1}{2}} frac{arcsin (x) ln (x)}{sqrt{1-x^{2}}} , dx \ &= - frac{pi^{2}}{9} ln 2 - 8 int_{0}^{frac{pi}{6}} u ln (sin u ) , du \ &= - 8 ln 2 int_{0}^{frac{pi}{6}} u , du - 8int_{0}^{frac{pi}{6}} u ln (sin u ) , du \ &= - 8 int_{0}^{frac{pi}{6}} u ln ( 2 sin u ) , du \ &= -8 text{Re} int_{0}^{frac{pi}{6}} u ln (1-e^{2iu}) , du \ &= 8 text{Re} int_{0}^{frac{pi}{6}} u sum_{n=1}^{infty} frac{e^{2in u}}{n} , du \ &= 8 sum_{n=1}^{infty} frac{1}{n} int_{0}^{frac{pi}{6}} u cos (2nu) , du \ &= frac{2 pi}{3} sum_{n=1}^{infty} frac{sin (frac{n pi}{3})}{n^{2}} + 2 sum_{n=1}^{infty} frac{cos (frac{n pi}{3})}{n^{3}} - 2 zeta(3) \ &= frac{2 pi}{3} Bigg( frac{sqrt{3}}{2} sum_{n=0}^{infty} frac{1}{(6n+1)^{2}} + frac{sqrt{3}}{2} sum_{n=0}^{infty} frac{1}{(6n+2)^{2}} - frac{sqrt{3}}{2} sum_{n=0}^{infty} frac{1}{(6n+4)^{2}} - frac{sqrt{3}}{2} sum_{n=0}^{infty} frac{1}{(6n+5)^{2}} Bigg) \ &+ 2 Bigg( frac{1}{2} sum_{n=0}^{infty} frac{1}{(6n+1)^{3}} - frac{1}{2} sum_{n=0}^{infty} frac{1}{(6n+2)^{3}} - sum_{n=0}^{infty} frac{1}{(6n+3)^{3}} - frac{1}{2} sum_{n=0}^{infty} frac{1}{(6n+4)^{3}} \ &+ frac{1}{2} sum_{n=0}^{infty} frac{1}{(6n+5)^{3}} + sum_{n=1}^{infty} frac{1}{(6n)^{3}} Bigg) - 2 zeta(3) \ &= frac{pi sqrt{3}}{108} Bigg( psi_{1} left(frac{1}{6} right) + psi_{1} left(frac{1}{3} right) - psi_{1} left(frac{2}{3} right) - psi_{1}left(frac{5}{6} right) Bigg) + frac{1}{432} Bigg( - psi_{2} left(frac{1}{6} right) + psi_{2} left(frac{1}{3} right) \ &-28 zeta(3) + psi_{2} left(frac{2}{3} right) - psi_{2} left(frac{5}{6} right) + 4 zeta(3)Bigg) - 2 zeta (3) .end{align}$$



From here I've been going in circles trying to get the answer in the form given above.





EDIT:



Using the duplication formula for the trigamma function (i.e., $ displaystyle 4 psi_{1}(2x) = psi_{1}(x) + psi_{1} left(x + frac{1}{2} right) $),



$$ begin{align} &psi_{1} left(frac{1}{6} right) + psi_{1} left(frac{1}{3} right) - psi_{1} left(frac{2}{3} right) - psi_{1}left(frac{5}{6} right) \ &= 4 psi_{1} left(frac{1}{3} right) -psi_{1} left(frac{2}{3} right) + psi_{1} left(frac{1}{3} right) -psi_{1} left(frac{2}{3} right) - 4 psi_{1} left(frac{2}{3} right) + psi_{1} left(frac{1}{3} right) \ &= 6 psi_{1} left(frac{1}{3} right) - 6 psi_{1} left(frac{2}{3} right). end{align}$$



Therefore,



$$ begin{align} sum_{n=1}^{infty} frac{1}{n^{3} binom{2n}{n}} &= frac{sqrt{3} pi}{18} Bigg( psi_{1} left(frac{1}{3} right) - psi_{1} left(frac{2}{3} right) Bigg) +
frac{1}{432} Bigg( - psi_{2} left(frac{1}{6} right) + psi_{2} left(frac{1}{3} right) \ &+ psi_{2} left(frac{2}{3} right) - psi_{2} left(frac{5}{6} right) - 2 zeta(3)Bigg) - 2 zeta (3). end{align}$$



So it comes down to somehow showing that $$-psi_{2} left(frac{1}{6} right) + psi_{2} left(frac{1}{3} right) + psi_{2} left(frac{2}{3} right) - psi_{2} left(frac{5}{6} right) = 312 zeta(3) .$$



SECOND EDIT:



Using the duplication formula for $psi_{2}(x)$, we get



$$ begin{align} &-psi_{2} left(frac{1}{6} right) + psi_{2} left(frac{1}{3} right) + psi_{2} left(frac{2}{3} right) - psi_{2} left(frac{5}{6} right) \ &= -8 psi_{2} left(frac{1}{3} right) +psi_{2} left(frac{2}{3} right) + psi_{2} left(frac{1}{3} right) +psi_{2} left(frac{2}{3} right) - 8 psi_{2} left(frac{2}{3} right) + psi_{2} left(frac{1}{3} right) \ &= -6 psi_{2} left(frac{1}{3} right) - 6 psi_{2} left(frac{2}{3} right) . end{align}$$



And using the more general multiplication formula, we get



$$ psi_{2} left(frac{1}{3} right) + psi_{2} left( frac{2}{3} right) + psi_{2}(1) = 27 psi_{2} (1) .$$



Therefore,



$$ -6 psi_{2} left(frac{1}{3} right) - 6 psi_{2} left(frac{2}{3} right) = -156 psi_{2} (1) = 312 zeta(3) .$$










share|cite|improve this question











$endgroup$




Wolfram MathWorld states that $$ sum_{n=1}^{infty} frac{1}{n^{3} binom{2n}{n}} = frac{ pi sqrt{3}}{18} Bigg[ psi_{1} left(frac{1}{3} right) - psi_{1} left(frac{2}{3} right) Bigg]- frac{4}{3} zeta(3) , , $$



where $psi_{1}(x)$ is the trigamma function.



I can't seem to get the answer in that form.



Using the Taylor expansion $ displaystyle arcsin^{2}(x) = frac{1}{2} sum_{n=1}^{infty} frac{1}{n^{2} binom{2n}{n}} (2x)^{2n}$, I get



$$ sum_{n=1}^{infty} frac{1}{n^{3} binom{2n}{n}} = 4 int_{0}^{frac{1}{2}} frac{arcsin^{2}(x)}{x} , dx. $$



Then I integrating by parts, I get



$$ begin{align} &4 int_{0}^{frac{1}{2}} frac{arcsin^{2}(x)}{x} , dx \ &= frac{pi^{2}}{9} ln left(frac{1}{2} right) - 8 int_{0}^{frac{1}{2}} frac{arcsin (x) ln (x)}{sqrt{1-x^{2}}} , dx \ &= - frac{pi^{2}}{9} ln 2 - 8 int_{0}^{frac{pi}{6}} u ln (sin u ) , du \ &= - 8 ln 2 int_{0}^{frac{pi}{6}} u , du - 8int_{0}^{frac{pi}{6}} u ln (sin u ) , du \ &= - 8 int_{0}^{frac{pi}{6}} u ln ( 2 sin u ) , du \ &= -8 text{Re} int_{0}^{frac{pi}{6}} u ln (1-e^{2iu}) , du \ &= 8 text{Re} int_{0}^{frac{pi}{6}} u sum_{n=1}^{infty} frac{e^{2in u}}{n} , du \ &= 8 sum_{n=1}^{infty} frac{1}{n} int_{0}^{frac{pi}{6}} u cos (2nu) , du \ &= frac{2 pi}{3} sum_{n=1}^{infty} frac{sin (frac{n pi}{3})}{n^{2}} + 2 sum_{n=1}^{infty} frac{cos (frac{n pi}{3})}{n^{3}} - 2 zeta(3) \ &= frac{2 pi}{3} Bigg( frac{sqrt{3}}{2} sum_{n=0}^{infty} frac{1}{(6n+1)^{2}} + frac{sqrt{3}}{2} sum_{n=0}^{infty} frac{1}{(6n+2)^{2}} - frac{sqrt{3}}{2} sum_{n=0}^{infty} frac{1}{(6n+4)^{2}} - frac{sqrt{3}}{2} sum_{n=0}^{infty} frac{1}{(6n+5)^{2}} Bigg) \ &+ 2 Bigg( frac{1}{2} sum_{n=0}^{infty} frac{1}{(6n+1)^{3}} - frac{1}{2} sum_{n=0}^{infty} frac{1}{(6n+2)^{3}} - sum_{n=0}^{infty} frac{1}{(6n+3)^{3}} - frac{1}{2} sum_{n=0}^{infty} frac{1}{(6n+4)^{3}} \ &+ frac{1}{2} sum_{n=0}^{infty} frac{1}{(6n+5)^{3}} + sum_{n=1}^{infty} frac{1}{(6n)^{3}} Bigg) - 2 zeta(3) \ &= frac{pi sqrt{3}}{108} Bigg( psi_{1} left(frac{1}{6} right) + psi_{1} left(frac{1}{3} right) - psi_{1} left(frac{2}{3} right) - psi_{1}left(frac{5}{6} right) Bigg) + frac{1}{432} Bigg( - psi_{2} left(frac{1}{6} right) + psi_{2} left(frac{1}{3} right) \ &-28 zeta(3) + psi_{2} left(frac{2}{3} right) - psi_{2} left(frac{5}{6} right) + 4 zeta(3)Bigg) - 2 zeta (3) .end{align}$$



From here I've been going in circles trying to get the answer in the form given above.





EDIT:



Using the duplication formula for the trigamma function (i.e., $ displaystyle 4 psi_{1}(2x) = psi_{1}(x) + psi_{1} left(x + frac{1}{2} right) $),



$$ begin{align} &psi_{1} left(frac{1}{6} right) + psi_{1} left(frac{1}{3} right) - psi_{1} left(frac{2}{3} right) - psi_{1}left(frac{5}{6} right) \ &= 4 psi_{1} left(frac{1}{3} right) -psi_{1} left(frac{2}{3} right) + psi_{1} left(frac{1}{3} right) -psi_{1} left(frac{2}{3} right) - 4 psi_{1} left(frac{2}{3} right) + psi_{1} left(frac{1}{3} right) \ &= 6 psi_{1} left(frac{1}{3} right) - 6 psi_{1} left(frac{2}{3} right). end{align}$$



Therefore,



$$ begin{align} sum_{n=1}^{infty} frac{1}{n^{3} binom{2n}{n}} &= frac{sqrt{3} pi}{18} Bigg( psi_{1} left(frac{1}{3} right) - psi_{1} left(frac{2}{3} right) Bigg) +
frac{1}{432} Bigg( - psi_{2} left(frac{1}{6} right) + psi_{2} left(frac{1}{3} right) \ &+ psi_{2} left(frac{2}{3} right) - psi_{2} left(frac{5}{6} right) - 2 zeta(3)Bigg) - 2 zeta (3). end{align}$$



So it comes down to somehow showing that $$-psi_{2} left(frac{1}{6} right) + psi_{2} left(frac{1}{3} right) + psi_{2} left(frac{2}{3} right) - psi_{2} left(frac{5}{6} right) = 312 zeta(3) .$$



SECOND EDIT:



Using the duplication formula for $psi_{2}(x)$, we get



$$ begin{align} &-psi_{2} left(frac{1}{6} right) + psi_{2} left(frac{1}{3} right) + psi_{2} left(frac{2}{3} right) - psi_{2} left(frac{5}{6} right) \ &= -8 psi_{2} left(frac{1}{3} right) +psi_{2} left(frac{2}{3} right) + psi_{2} left(frac{1}{3} right) +psi_{2} left(frac{2}{3} right) - 8 psi_{2} left(frac{2}{3} right) + psi_{2} left(frac{1}{3} right) \ &= -6 psi_{2} left(frac{1}{3} right) - 6 psi_{2} left(frac{2}{3} right) . end{align}$$



And using the more general multiplication formula, we get



$$ psi_{2} left(frac{1}{3} right) + psi_{2} left( frac{2}{3} right) + psi_{2}(1) = 27 psi_{2} (1) .$$



Therefore,



$$ -6 psi_{2} left(frac{1}{3} right) - 6 psi_{2} left(frac{2}{3} right) = -156 psi_{2} (1) = 312 zeta(3) .$$







sequences-and-series complex-analysis special-functions binomial-coefficients gamma-function






share|cite|improve this question















share|cite|improve this question













share|cite|improve this question




share|cite|improve this question








edited Dec 25 '18 at 10:30









Did

249k23229468




249k23229468










asked Sep 20 '13 at 21:20









Random VariableRandom Variable

25.6k173139




25.6k173139












  • $begingroup$
    Isn't this the sum that came up in Apery's proof of the irrationality of $zeta(3)$? If so, it may be in Alf van der Poorten's paper, "A Proof that Euler Missed". Also, I think the sum may be done in Comtet's book on Combinatorics. Also, look at the "Related" questions running down the side of this page.
    $endgroup$
    – Gerry Myerson
    Sep 21 '13 at 1:15












  • $begingroup$
    That sum is the alternating version of this one.
    $endgroup$
    – Random Variable
    Sep 21 '13 at 1:31










  • $begingroup$
    And the techniques that apply to the alternating version don't work for the one you ask about?
    $endgroup$
    – Gerry Myerson
    Sep 21 '13 at 1:41










  • $begingroup$
    They work to a point. But the integrand of the other one is $ displaystyle frac{text{arcsinh}^{2}(x)}{x}$.
    $endgroup$
    – Random Variable
    Sep 21 '13 at 1:56












  • $begingroup$
    use Clausen function
    $endgroup$
    – alexjo
    Nov 13 '13 at 21:08


















  • $begingroup$
    Isn't this the sum that came up in Apery's proof of the irrationality of $zeta(3)$? If so, it may be in Alf van der Poorten's paper, "A Proof that Euler Missed". Also, I think the sum may be done in Comtet's book on Combinatorics. Also, look at the "Related" questions running down the side of this page.
    $endgroup$
    – Gerry Myerson
    Sep 21 '13 at 1:15












  • $begingroup$
    That sum is the alternating version of this one.
    $endgroup$
    – Random Variable
    Sep 21 '13 at 1:31










  • $begingroup$
    And the techniques that apply to the alternating version don't work for the one you ask about?
    $endgroup$
    – Gerry Myerson
    Sep 21 '13 at 1:41










  • $begingroup$
    They work to a point. But the integrand of the other one is $ displaystyle frac{text{arcsinh}^{2}(x)}{x}$.
    $endgroup$
    – Random Variable
    Sep 21 '13 at 1:56












  • $begingroup$
    use Clausen function
    $endgroup$
    – alexjo
    Nov 13 '13 at 21:08
















$begingroup$
Isn't this the sum that came up in Apery's proof of the irrationality of $zeta(3)$? If so, it may be in Alf van der Poorten's paper, "A Proof that Euler Missed". Also, I think the sum may be done in Comtet's book on Combinatorics. Also, look at the "Related" questions running down the side of this page.
$endgroup$
– Gerry Myerson
Sep 21 '13 at 1:15






$begingroup$
Isn't this the sum that came up in Apery's proof of the irrationality of $zeta(3)$? If so, it may be in Alf van der Poorten's paper, "A Proof that Euler Missed". Also, I think the sum may be done in Comtet's book on Combinatorics. Also, look at the "Related" questions running down the side of this page.
$endgroup$
– Gerry Myerson
Sep 21 '13 at 1:15














$begingroup$
That sum is the alternating version of this one.
$endgroup$
– Random Variable
Sep 21 '13 at 1:31




$begingroup$
That sum is the alternating version of this one.
$endgroup$
– Random Variable
Sep 21 '13 at 1:31












$begingroup$
And the techniques that apply to the alternating version don't work for the one you ask about?
$endgroup$
– Gerry Myerson
Sep 21 '13 at 1:41




$begingroup$
And the techniques that apply to the alternating version don't work for the one you ask about?
$endgroup$
– Gerry Myerson
Sep 21 '13 at 1:41












$begingroup$
They work to a point. But the integrand of the other one is $ displaystyle frac{text{arcsinh}^{2}(x)}{x}$.
$endgroup$
– Random Variable
Sep 21 '13 at 1:56






$begingroup$
They work to a point. But the integrand of the other one is $ displaystyle frac{text{arcsinh}^{2}(x)}{x}$.
$endgroup$
– Random Variable
Sep 21 '13 at 1:56














$begingroup$
use Clausen function
$endgroup$
– alexjo
Nov 13 '13 at 21:08




$begingroup$
use Clausen function
$endgroup$
– alexjo
Nov 13 '13 at 21:08










2 Answers
2






active

oldest

votes


















14












$begingroup$

Using the Taylor expansion for $arcsin^2(x)$ and integrating, it's easy to show (as you made) that
$$
begin{align}
sum_{n=1}^{infty} frac{1}{n^{3} binom{2n}{n}}
&= 4 int_{0}^{frac{1}{2}} frac{arcsin^{2}(x)}{x} operatorname{d}x = - 8 int_{0}^{frac{pi}{6}} u ln ( 2 sin u ) operatorname{d}u \
&= frac{2 pi}{3} sum_{n=1}^{infty} frac{sin (frac{n pi}{3})}{n^{2}} + 2 sum_{n=1}^{infty} frac{cos (frac{n pi}{3})}{n^{3}} - 2 zeta(3)\
&=frac{2 pi}{3}operatorname{Cl}_2left(frac{pi}{3}right)+2operatorname{Cl}_3left(frac{pi}{3}right)-2 zeta(3)
end{align}
$$
recalling that the Clausen function are defined as
$$
operatorname{Cl}_{m}(theta)=
begin{cases}displaystyle
sum_{n=1}^{infty} frac{sin (ntheta)}{n^{m}}& text{for } mtext{ even}\
displaystylesum_{n=1}^{infty} frac{cos(ntheta)}{n^{m}}& text{for } mtext{ odd}
end{cases}
$$
The value of the Clausen function $operatorname{Cl}_3$ at $frac{pi}{3}$ is
$$
operatorname{Cl}_3left(frac{pi}{3}right)=frac{1}{2}left(1-2^{-2}right)left(1-3^{-2}right)zeta(3)=frac{1}{3}zeta(3)
$$
From the duplication formula
$$
operatorname{Cl}_{2m}(2theta)=2^{2m-1}left[operatorname{Cl}_{2m}(theta)-operatorname{Cl}_{2m}(pi-theta)right]
$$
we find
$$
operatorname{Cl}_{2}left(frac{2pi}{3}right)=frac{2}{3}operatorname{Cl}_{2}left(frac{pi}{3}right).
$$
From the identities for the trigamma function at $1/3$ and $2/3$
$$
begin{align}
psi_1left(frac{1}{3}right) &=frac{2pi^2}{3}+3sqrt 3operatorname{Cl}_{2}left(frac{2pi}{3}right)\
psi_1left(frac{2}{3}right) &=frac{2pi^2}{3}-3sqrt 3operatorname{Cl}_{2}left(frac{2pi}{3}right)
end{align}
$$
one has
$$
psi_1left(frac{1}{3}right)-psi_1left(frac{2}{3}right)=6sqrt 3operatorname{Cl}_{2}left(frac{2pi}{3}right)
$$
and then
$$
operatorname{Cl}_{2}left(frac{pi}{3}right)=frac{sqrt 3}{12}left[psi_1left(frac{1}{3}right)-psi_1left(frac{2}{3}right)right]
$$



Finally, putting all together, we have
$$
begin{align}
sum_{n=1}^{infty} frac{1}{n^{3} binom{2n}{n}}
&=frac{2 pi}{3}operatorname{Cl}_2left(frac{pi}{3}right)+2operatorname{Cl}_3left(frac{pi}{3}right)-2 zeta(3)\
&=frac{2 pi}{3}frac{sqrt 3}{12}left[psi_1left(frac{1}{3}right)-psi_1left(frac{2}{3}right)right]+2frac{1}{3}zeta(3)-2zeta(3)\
&=frac{pisqrt 3}{18}left[psi_1left(frac{1}{3}right)-psi_1left(frac{2}{3}right)right]-frac{4}{3}zeta(3).
end{align}
$$






share|cite|improve this answer









$endgroup$





















    9












    $begingroup$

    $newcommand{angles}[1]{leftlangle, #1 ,rightrangle}
    newcommand{braces}[1]{leftlbrace, #1 ,rightrbrace}
    newcommand{bracks}[1]{leftlbrack, #1 ,rightrbrack}
    newcommand{ceil}[1]{,leftlceil, #1 ,rightrceil,}
    newcommand{dd}{{rm d}}
    newcommand{ds}[1]{displaystyle{#1}}
    newcommand{expo}[1]{,{rm e}^{#1},}
    newcommand{fermi}{,{rm f}}
    newcommand{floor}[1]{,leftlfloor #1 rightrfloor,}
    newcommand{half}{{1 over 2}}
    newcommand{ic}{{rm i}}
    newcommand{iff}{Longleftrightarrow}
    newcommand{imp}{Longrightarrow}
    newcommand{pars}[1]{left(, #1 ,right)}
    newcommand{partiald}[3]{frac{partial^{#1} #2}{partial #3^{#1}}}
    newcommand{pp}{{cal P}}
    newcommand{root}[2]{,sqrt[#1]{vphantom{large A},#2,},}
    newcommand{sech}{,{rm sech}}
    newcommand{sgn}{,{rm sgn}}
    newcommand{totald}[3]{frac{{rm d}^{#1} #2}{{rm d} #3^{#1}}}
    newcommand{verts}[1]{leftvert, #1 ,rightvert}$

    $ds{sum_{n=1}^{infty} frac{1}{n^{3} binom{2n}{n}} = frac{ pi sqrt{3}}{18} Big[ psi_{1} left(frac{1}{3} right) - psi_{1} left(frac{2}{3} right) Big]- frac{4}{3} zeta(3)}$




    begin{align}
    &sum_{n = 1}^{infty}{x^{n} over n{2n choose n}}
    =sum_{n = 1}^{infty}x^{n},
    {Gammapars{n}Gammapars{n + 1} over Gammapars{2n + 1}}
    =sum_{n = 1}^{infty}x^{n}int_{0}^{1}t^{n - 1}pars{1 - t}^{n},dd t
    \[3mm] & =
    int_{0}^{1}sum_{n = 1}^{infty}bracks{txpars{1 - t}}^{n}
    ,{dd t over t}
    =int_{0}^{1}bracks{{1 over 1 - txpars{1 - t}} - 1},{dd t over t}
    \[3mm] & =
    int_{0}^{1}{xpars{1 - t} over 1 - txpars{1 - t}},dd t
    end{align}




    begin{align}
    &sum_{n = 1}^{infty}{x^{n} over n^{2}{2n choose n}}
    =int_{0}^{x}dd yint_{0}^{1}{1 - t over 1 - typars{1 - t}},dd t
    =-int_{0}^{1}{lnpars{1 - bracks{1 - t}tx} over t},dd t
    end{align}




    begin{align}
    &color{#66f}{largesum_{n = 1}^{infty}{1 over n^{3}{2n choose n}}}
    =-int_{0}^{1}{dd x over x}
    int_{0}^{1}{lnpars{1 - bracks{1 - t}tx} over t},dd t
    \[3mm] = &
    int_{0}^{1}
    {{rm Li}_{2}pars{tbracks{1 - t}} over t},dd t
    \[3mm] = &
    color{#66f}{{pi over 36root{3}}, left[psi ^{(1)}left(frac{1}{3}right)-psi ^{(1)}left(frac{2}{3}right)+psi ^{(1)}left(frac{1}{6}right)-psi ^{(1)}left(frac{5}{6}right)right]-frac{4 zeta (3)}{3}}
    \[3mm] approx & {tt 0.5229}
    end{align}







    share|cite|improve this answer











    $endgroup$














      Your Answer








      StackExchange.ready(function() {
      var channelOptions = {
      tags: "".split(" "),
      id: "69"
      };
      initTagRenderer("".split(" "), "".split(" "), channelOptions);

      StackExchange.using("externalEditor", function() {
      // Have to fire editor after snippets, if snippets enabled
      if (StackExchange.settings.snippets.snippetsEnabled) {
      StackExchange.using("snippets", function() {
      createEditor();
      });
      }
      else {
      createEditor();
      }
      });

      function createEditor() {
      StackExchange.prepareEditor({
      heartbeatType: 'answer',
      autoActivateHeartbeat: false,
      convertImagesToLinks: true,
      noModals: true,
      showLowRepImageUploadWarning: true,
      reputationToPostImages: 10,
      bindNavPrevention: true,
      postfix: "",
      imageUploader: {
      brandingHtml: "Powered by u003ca class="icon-imgur-white" href="https://imgur.com/"u003eu003c/au003e",
      contentPolicyHtml: "User contributions licensed under u003ca href="https://creativecommons.org/licenses/by-sa/3.0/"u003ecc by-sa 3.0 with attribution requiredu003c/au003e u003ca href="https://stackoverflow.com/legal/content-policy"u003e(content policy)u003c/au003e",
      allowUrls: true
      },
      noCode: true, onDemand: true,
      discardSelector: ".discard-answer"
      ,immediatelyShowMarkdownHelp:true
      });


      }
      });














      draft saved

      draft discarded


















      StackExchange.ready(
      function () {
      StackExchange.openid.initPostLogin('.new-post-login', 'https%3a%2f%2fmath.stackexchange.com%2fquestions%2f499926%2fevaluating-the-sum-of-the-series-sum-limits-n-1-infty-frac1n3-bin%23new-answer', 'question_page');
      }
      );

      Post as a guest















      Required, but never shown

























      2 Answers
      2






      active

      oldest

      votes








      2 Answers
      2






      active

      oldest

      votes









      active

      oldest

      votes






      active

      oldest

      votes









      14












      $begingroup$

      Using the Taylor expansion for $arcsin^2(x)$ and integrating, it's easy to show (as you made) that
      $$
      begin{align}
      sum_{n=1}^{infty} frac{1}{n^{3} binom{2n}{n}}
      &= 4 int_{0}^{frac{1}{2}} frac{arcsin^{2}(x)}{x} operatorname{d}x = - 8 int_{0}^{frac{pi}{6}} u ln ( 2 sin u ) operatorname{d}u \
      &= frac{2 pi}{3} sum_{n=1}^{infty} frac{sin (frac{n pi}{3})}{n^{2}} + 2 sum_{n=1}^{infty} frac{cos (frac{n pi}{3})}{n^{3}} - 2 zeta(3)\
      &=frac{2 pi}{3}operatorname{Cl}_2left(frac{pi}{3}right)+2operatorname{Cl}_3left(frac{pi}{3}right)-2 zeta(3)
      end{align}
      $$
      recalling that the Clausen function are defined as
      $$
      operatorname{Cl}_{m}(theta)=
      begin{cases}displaystyle
      sum_{n=1}^{infty} frac{sin (ntheta)}{n^{m}}& text{for } mtext{ even}\
      displaystylesum_{n=1}^{infty} frac{cos(ntheta)}{n^{m}}& text{for } mtext{ odd}
      end{cases}
      $$
      The value of the Clausen function $operatorname{Cl}_3$ at $frac{pi}{3}$ is
      $$
      operatorname{Cl}_3left(frac{pi}{3}right)=frac{1}{2}left(1-2^{-2}right)left(1-3^{-2}right)zeta(3)=frac{1}{3}zeta(3)
      $$
      From the duplication formula
      $$
      operatorname{Cl}_{2m}(2theta)=2^{2m-1}left[operatorname{Cl}_{2m}(theta)-operatorname{Cl}_{2m}(pi-theta)right]
      $$
      we find
      $$
      operatorname{Cl}_{2}left(frac{2pi}{3}right)=frac{2}{3}operatorname{Cl}_{2}left(frac{pi}{3}right).
      $$
      From the identities for the trigamma function at $1/3$ and $2/3$
      $$
      begin{align}
      psi_1left(frac{1}{3}right) &=frac{2pi^2}{3}+3sqrt 3operatorname{Cl}_{2}left(frac{2pi}{3}right)\
      psi_1left(frac{2}{3}right) &=frac{2pi^2}{3}-3sqrt 3operatorname{Cl}_{2}left(frac{2pi}{3}right)
      end{align}
      $$
      one has
      $$
      psi_1left(frac{1}{3}right)-psi_1left(frac{2}{3}right)=6sqrt 3operatorname{Cl}_{2}left(frac{2pi}{3}right)
      $$
      and then
      $$
      operatorname{Cl}_{2}left(frac{pi}{3}right)=frac{sqrt 3}{12}left[psi_1left(frac{1}{3}right)-psi_1left(frac{2}{3}right)right]
      $$



      Finally, putting all together, we have
      $$
      begin{align}
      sum_{n=1}^{infty} frac{1}{n^{3} binom{2n}{n}}
      &=frac{2 pi}{3}operatorname{Cl}_2left(frac{pi}{3}right)+2operatorname{Cl}_3left(frac{pi}{3}right)-2 zeta(3)\
      &=frac{2 pi}{3}frac{sqrt 3}{12}left[psi_1left(frac{1}{3}right)-psi_1left(frac{2}{3}right)right]+2frac{1}{3}zeta(3)-2zeta(3)\
      &=frac{pisqrt 3}{18}left[psi_1left(frac{1}{3}right)-psi_1left(frac{2}{3}right)right]-frac{4}{3}zeta(3).
      end{align}
      $$






      share|cite|improve this answer









      $endgroup$


















        14












        $begingroup$

        Using the Taylor expansion for $arcsin^2(x)$ and integrating, it's easy to show (as you made) that
        $$
        begin{align}
        sum_{n=1}^{infty} frac{1}{n^{3} binom{2n}{n}}
        &= 4 int_{0}^{frac{1}{2}} frac{arcsin^{2}(x)}{x} operatorname{d}x = - 8 int_{0}^{frac{pi}{6}} u ln ( 2 sin u ) operatorname{d}u \
        &= frac{2 pi}{3} sum_{n=1}^{infty} frac{sin (frac{n pi}{3})}{n^{2}} + 2 sum_{n=1}^{infty} frac{cos (frac{n pi}{3})}{n^{3}} - 2 zeta(3)\
        &=frac{2 pi}{3}operatorname{Cl}_2left(frac{pi}{3}right)+2operatorname{Cl}_3left(frac{pi}{3}right)-2 zeta(3)
        end{align}
        $$
        recalling that the Clausen function are defined as
        $$
        operatorname{Cl}_{m}(theta)=
        begin{cases}displaystyle
        sum_{n=1}^{infty} frac{sin (ntheta)}{n^{m}}& text{for } mtext{ even}\
        displaystylesum_{n=1}^{infty} frac{cos(ntheta)}{n^{m}}& text{for } mtext{ odd}
        end{cases}
        $$
        The value of the Clausen function $operatorname{Cl}_3$ at $frac{pi}{3}$ is
        $$
        operatorname{Cl}_3left(frac{pi}{3}right)=frac{1}{2}left(1-2^{-2}right)left(1-3^{-2}right)zeta(3)=frac{1}{3}zeta(3)
        $$
        From the duplication formula
        $$
        operatorname{Cl}_{2m}(2theta)=2^{2m-1}left[operatorname{Cl}_{2m}(theta)-operatorname{Cl}_{2m}(pi-theta)right]
        $$
        we find
        $$
        operatorname{Cl}_{2}left(frac{2pi}{3}right)=frac{2}{3}operatorname{Cl}_{2}left(frac{pi}{3}right).
        $$
        From the identities for the trigamma function at $1/3$ and $2/3$
        $$
        begin{align}
        psi_1left(frac{1}{3}right) &=frac{2pi^2}{3}+3sqrt 3operatorname{Cl}_{2}left(frac{2pi}{3}right)\
        psi_1left(frac{2}{3}right) &=frac{2pi^2}{3}-3sqrt 3operatorname{Cl}_{2}left(frac{2pi}{3}right)
        end{align}
        $$
        one has
        $$
        psi_1left(frac{1}{3}right)-psi_1left(frac{2}{3}right)=6sqrt 3operatorname{Cl}_{2}left(frac{2pi}{3}right)
        $$
        and then
        $$
        operatorname{Cl}_{2}left(frac{pi}{3}right)=frac{sqrt 3}{12}left[psi_1left(frac{1}{3}right)-psi_1left(frac{2}{3}right)right]
        $$



        Finally, putting all together, we have
        $$
        begin{align}
        sum_{n=1}^{infty} frac{1}{n^{3} binom{2n}{n}}
        &=frac{2 pi}{3}operatorname{Cl}_2left(frac{pi}{3}right)+2operatorname{Cl}_3left(frac{pi}{3}right)-2 zeta(3)\
        &=frac{2 pi}{3}frac{sqrt 3}{12}left[psi_1left(frac{1}{3}right)-psi_1left(frac{2}{3}right)right]+2frac{1}{3}zeta(3)-2zeta(3)\
        &=frac{pisqrt 3}{18}left[psi_1left(frac{1}{3}right)-psi_1left(frac{2}{3}right)right]-frac{4}{3}zeta(3).
        end{align}
        $$






        share|cite|improve this answer









        $endgroup$
















          14












          14








          14





          $begingroup$

          Using the Taylor expansion for $arcsin^2(x)$ and integrating, it's easy to show (as you made) that
          $$
          begin{align}
          sum_{n=1}^{infty} frac{1}{n^{3} binom{2n}{n}}
          &= 4 int_{0}^{frac{1}{2}} frac{arcsin^{2}(x)}{x} operatorname{d}x = - 8 int_{0}^{frac{pi}{6}} u ln ( 2 sin u ) operatorname{d}u \
          &= frac{2 pi}{3} sum_{n=1}^{infty} frac{sin (frac{n pi}{3})}{n^{2}} + 2 sum_{n=1}^{infty} frac{cos (frac{n pi}{3})}{n^{3}} - 2 zeta(3)\
          &=frac{2 pi}{3}operatorname{Cl}_2left(frac{pi}{3}right)+2operatorname{Cl}_3left(frac{pi}{3}right)-2 zeta(3)
          end{align}
          $$
          recalling that the Clausen function are defined as
          $$
          operatorname{Cl}_{m}(theta)=
          begin{cases}displaystyle
          sum_{n=1}^{infty} frac{sin (ntheta)}{n^{m}}& text{for } mtext{ even}\
          displaystylesum_{n=1}^{infty} frac{cos(ntheta)}{n^{m}}& text{for } mtext{ odd}
          end{cases}
          $$
          The value of the Clausen function $operatorname{Cl}_3$ at $frac{pi}{3}$ is
          $$
          operatorname{Cl}_3left(frac{pi}{3}right)=frac{1}{2}left(1-2^{-2}right)left(1-3^{-2}right)zeta(3)=frac{1}{3}zeta(3)
          $$
          From the duplication formula
          $$
          operatorname{Cl}_{2m}(2theta)=2^{2m-1}left[operatorname{Cl}_{2m}(theta)-operatorname{Cl}_{2m}(pi-theta)right]
          $$
          we find
          $$
          operatorname{Cl}_{2}left(frac{2pi}{3}right)=frac{2}{3}operatorname{Cl}_{2}left(frac{pi}{3}right).
          $$
          From the identities for the trigamma function at $1/3$ and $2/3$
          $$
          begin{align}
          psi_1left(frac{1}{3}right) &=frac{2pi^2}{3}+3sqrt 3operatorname{Cl}_{2}left(frac{2pi}{3}right)\
          psi_1left(frac{2}{3}right) &=frac{2pi^2}{3}-3sqrt 3operatorname{Cl}_{2}left(frac{2pi}{3}right)
          end{align}
          $$
          one has
          $$
          psi_1left(frac{1}{3}right)-psi_1left(frac{2}{3}right)=6sqrt 3operatorname{Cl}_{2}left(frac{2pi}{3}right)
          $$
          and then
          $$
          operatorname{Cl}_{2}left(frac{pi}{3}right)=frac{sqrt 3}{12}left[psi_1left(frac{1}{3}right)-psi_1left(frac{2}{3}right)right]
          $$



          Finally, putting all together, we have
          $$
          begin{align}
          sum_{n=1}^{infty} frac{1}{n^{3} binom{2n}{n}}
          &=frac{2 pi}{3}operatorname{Cl}_2left(frac{pi}{3}right)+2operatorname{Cl}_3left(frac{pi}{3}right)-2 zeta(3)\
          &=frac{2 pi}{3}frac{sqrt 3}{12}left[psi_1left(frac{1}{3}right)-psi_1left(frac{2}{3}right)right]+2frac{1}{3}zeta(3)-2zeta(3)\
          &=frac{pisqrt 3}{18}left[psi_1left(frac{1}{3}right)-psi_1left(frac{2}{3}right)right]-frac{4}{3}zeta(3).
          end{align}
          $$






          share|cite|improve this answer









          $endgroup$



          Using the Taylor expansion for $arcsin^2(x)$ and integrating, it's easy to show (as you made) that
          $$
          begin{align}
          sum_{n=1}^{infty} frac{1}{n^{3} binom{2n}{n}}
          &= 4 int_{0}^{frac{1}{2}} frac{arcsin^{2}(x)}{x} operatorname{d}x = - 8 int_{0}^{frac{pi}{6}} u ln ( 2 sin u ) operatorname{d}u \
          &= frac{2 pi}{3} sum_{n=1}^{infty} frac{sin (frac{n pi}{3})}{n^{2}} + 2 sum_{n=1}^{infty} frac{cos (frac{n pi}{3})}{n^{3}} - 2 zeta(3)\
          &=frac{2 pi}{3}operatorname{Cl}_2left(frac{pi}{3}right)+2operatorname{Cl}_3left(frac{pi}{3}right)-2 zeta(3)
          end{align}
          $$
          recalling that the Clausen function are defined as
          $$
          operatorname{Cl}_{m}(theta)=
          begin{cases}displaystyle
          sum_{n=1}^{infty} frac{sin (ntheta)}{n^{m}}& text{for } mtext{ even}\
          displaystylesum_{n=1}^{infty} frac{cos(ntheta)}{n^{m}}& text{for } mtext{ odd}
          end{cases}
          $$
          The value of the Clausen function $operatorname{Cl}_3$ at $frac{pi}{3}$ is
          $$
          operatorname{Cl}_3left(frac{pi}{3}right)=frac{1}{2}left(1-2^{-2}right)left(1-3^{-2}right)zeta(3)=frac{1}{3}zeta(3)
          $$
          From the duplication formula
          $$
          operatorname{Cl}_{2m}(2theta)=2^{2m-1}left[operatorname{Cl}_{2m}(theta)-operatorname{Cl}_{2m}(pi-theta)right]
          $$
          we find
          $$
          operatorname{Cl}_{2}left(frac{2pi}{3}right)=frac{2}{3}operatorname{Cl}_{2}left(frac{pi}{3}right).
          $$
          From the identities for the trigamma function at $1/3$ and $2/3$
          $$
          begin{align}
          psi_1left(frac{1}{3}right) &=frac{2pi^2}{3}+3sqrt 3operatorname{Cl}_{2}left(frac{2pi}{3}right)\
          psi_1left(frac{2}{3}right) &=frac{2pi^2}{3}-3sqrt 3operatorname{Cl}_{2}left(frac{2pi}{3}right)
          end{align}
          $$
          one has
          $$
          psi_1left(frac{1}{3}right)-psi_1left(frac{2}{3}right)=6sqrt 3operatorname{Cl}_{2}left(frac{2pi}{3}right)
          $$
          and then
          $$
          operatorname{Cl}_{2}left(frac{pi}{3}right)=frac{sqrt 3}{12}left[psi_1left(frac{1}{3}right)-psi_1left(frac{2}{3}right)right]
          $$



          Finally, putting all together, we have
          $$
          begin{align}
          sum_{n=1}^{infty} frac{1}{n^{3} binom{2n}{n}}
          &=frac{2 pi}{3}operatorname{Cl}_2left(frac{pi}{3}right)+2operatorname{Cl}_3left(frac{pi}{3}right)-2 zeta(3)\
          &=frac{2 pi}{3}frac{sqrt 3}{12}left[psi_1left(frac{1}{3}right)-psi_1left(frac{2}{3}right)right]+2frac{1}{3}zeta(3)-2zeta(3)\
          &=frac{pisqrt 3}{18}left[psi_1left(frac{1}{3}right)-psi_1left(frac{2}{3}right)right]-frac{4}{3}zeta(3).
          end{align}
          $$







          share|cite|improve this answer












          share|cite|improve this answer



          share|cite|improve this answer










          answered Nov 13 '13 at 3:09









          alexjoalexjo

          12.5k1430




          12.5k1430























              9












              $begingroup$

              $newcommand{angles}[1]{leftlangle, #1 ,rightrangle}
              newcommand{braces}[1]{leftlbrace, #1 ,rightrbrace}
              newcommand{bracks}[1]{leftlbrack, #1 ,rightrbrack}
              newcommand{ceil}[1]{,leftlceil, #1 ,rightrceil,}
              newcommand{dd}{{rm d}}
              newcommand{ds}[1]{displaystyle{#1}}
              newcommand{expo}[1]{,{rm e}^{#1},}
              newcommand{fermi}{,{rm f}}
              newcommand{floor}[1]{,leftlfloor #1 rightrfloor,}
              newcommand{half}{{1 over 2}}
              newcommand{ic}{{rm i}}
              newcommand{iff}{Longleftrightarrow}
              newcommand{imp}{Longrightarrow}
              newcommand{pars}[1]{left(, #1 ,right)}
              newcommand{partiald}[3]{frac{partial^{#1} #2}{partial #3^{#1}}}
              newcommand{pp}{{cal P}}
              newcommand{root}[2]{,sqrt[#1]{vphantom{large A},#2,},}
              newcommand{sech}{,{rm sech}}
              newcommand{sgn}{,{rm sgn}}
              newcommand{totald}[3]{frac{{rm d}^{#1} #2}{{rm d} #3^{#1}}}
              newcommand{verts}[1]{leftvert, #1 ,rightvert}$

              $ds{sum_{n=1}^{infty} frac{1}{n^{3} binom{2n}{n}} = frac{ pi sqrt{3}}{18} Big[ psi_{1} left(frac{1}{3} right) - psi_{1} left(frac{2}{3} right) Big]- frac{4}{3} zeta(3)}$




              begin{align}
              &sum_{n = 1}^{infty}{x^{n} over n{2n choose n}}
              =sum_{n = 1}^{infty}x^{n},
              {Gammapars{n}Gammapars{n + 1} over Gammapars{2n + 1}}
              =sum_{n = 1}^{infty}x^{n}int_{0}^{1}t^{n - 1}pars{1 - t}^{n},dd t
              \[3mm] & =
              int_{0}^{1}sum_{n = 1}^{infty}bracks{txpars{1 - t}}^{n}
              ,{dd t over t}
              =int_{0}^{1}bracks{{1 over 1 - txpars{1 - t}} - 1},{dd t over t}
              \[3mm] & =
              int_{0}^{1}{xpars{1 - t} over 1 - txpars{1 - t}},dd t
              end{align}




              begin{align}
              &sum_{n = 1}^{infty}{x^{n} over n^{2}{2n choose n}}
              =int_{0}^{x}dd yint_{0}^{1}{1 - t over 1 - typars{1 - t}},dd t
              =-int_{0}^{1}{lnpars{1 - bracks{1 - t}tx} over t},dd t
              end{align}




              begin{align}
              &color{#66f}{largesum_{n = 1}^{infty}{1 over n^{3}{2n choose n}}}
              =-int_{0}^{1}{dd x over x}
              int_{0}^{1}{lnpars{1 - bracks{1 - t}tx} over t},dd t
              \[3mm] = &
              int_{0}^{1}
              {{rm Li}_{2}pars{tbracks{1 - t}} over t},dd t
              \[3mm] = &
              color{#66f}{{pi over 36root{3}}, left[psi ^{(1)}left(frac{1}{3}right)-psi ^{(1)}left(frac{2}{3}right)+psi ^{(1)}left(frac{1}{6}right)-psi ^{(1)}left(frac{5}{6}right)right]-frac{4 zeta (3)}{3}}
              \[3mm] approx & {tt 0.5229}
              end{align}







              share|cite|improve this answer











              $endgroup$


















                9












                $begingroup$

                $newcommand{angles}[1]{leftlangle, #1 ,rightrangle}
                newcommand{braces}[1]{leftlbrace, #1 ,rightrbrace}
                newcommand{bracks}[1]{leftlbrack, #1 ,rightrbrack}
                newcommand{ceil}[1]{,leftlceil, #1 ,rightrceil,}
                newcommand{dd}{{rm d}}
                newcommand{ds}[1]{displaystyle{#1}}
                newcommand{expo}[1]{,{rm e}^{#1},}
                newcommand{fermi}{,{rm f}}
                newcommand{floor}[1]{,leftlfloor #1 rightrfloor,}
                newcommand{half}{{1 over 2}}
                newcommand{ic}{{rm i}}
                newcommand{iff}{Longleftrightarrow}
                newcommand{imp}{Longrightarrow}
                newcommand{pars}[1]{left(, #1 ,right)}
                newcommand{partiald}[3]{frac{partial^{#1} #2}{partial #3^{#1}}}
                newcommand{pp}{{cal P}}
                newcommand{root}[2]{,sqrt[#1]{vphantom{large A},#2,},}
                newcommand{sech}{,{rm sech}}
                newcommand{sgn}{,{rm sgn}}
                newcommand{totald}[3]{frac{{rm d}^{#1} #2}{{rm d} #3^{#1}}}
                newcommand{verts}[1]{leftvert, #1 ,rightvert}$

                $ds{sum_{n=1}^{infty} frac{1}{n^{3} binom{2n}{n}} = frac{ pi sqrt{3}}{18} Big[ psi_{1} left(frac{1}{3} right) - psi_{1} left(frac{2}{3} right) Big]- frac{4}{3} zeta(3)}$




                begin{align}
                &sum_{n = 1}^{infty}{x^{n} over n{2n choose n}}
                =sum_{n = 1}^{infty}x^{n},
                {Gammapars{n}Gammapars{n + 1} over Gammapars{2n + 1}}
                =sum_{n = 1}^{infty}x^{n}int_{0}^{1}t^{n - 1}pars{1 - t}^{n},dd t
                \[3mm] & =
                int_{0}^{1}sum_{n = 1}^{infty}bracks{txpars{1 - t}}^{n}
                ,{dd t over t}
                =int_{0}^{1}bracks{{1 over 1 - txpars{1 - t}} - 1},{dd t over t}
                \[3mm] & =
                int_{0}^{1}{xpars{1 - t} over 1 - txpars{1 - t}},dd t
                end{align}




                begin{align}
                &sum_{n = 1}^{infty}{x^{n} over n^{2}{2n choose n}}
                =int_{0}^{x}dd yint_{0}^{1}{1 - t over 1 - typars{1 - t}},dd t
                =-int_{0}^{1}{lnpars{1 - bracks{1 - t}tx} over t},dd t
                end{align}




                begin{align}
                &color{#66f}{largesum_{n = 1}^{infty}{1 over n^{3}{2n choose n}}}
                =-int_{0}^{1}{dd x over x}
                int_{0}^{1}{lnpars{1 - bracks{1 - t}tx} over t},dd t
                \[3mm] = &
                int_{0}^{1}
                {{rm Li}_{2}pars{tbracks{1 - t}} over t},dd t
                \[3mm] = &
                color{#66f}{{pi over 36root{3}}, left[psi ^{(1)}left(frac{1}{3}right)-psi ^{(1)}left(frac{2}{3}right)+psi ^{(1)}left(frac{1}{6}right)-psi ^{(1)}left(frac{5}{6}right)right]-frac{4 zeta (3)}{3}}
                \[3mm] approx & {tt 0.5229}
                end{align}







                share|cite|improve this answer











                $endgroup$
















                  9












                  9








                  9





                  $begingroup$

                  $newcommand{angles}[1]{leftlangle, #1 ,rightrangle}
                  newcommand{braces}[1]{leftlbrace, #1 ,rightrbrace}
                  newcommand{bracks}[1]{leftlbrack, #1 ,rightrbrack}
                  newcommand{ceil}[1]{,leftlceil, #1 ,rightrceil,}
                  newcommand{dd}{{rm d}}
                  newcommand{ds}[1]{displaystyle{#1}}
                  newcommand{expo}[1]{,{rm e}^{#1},}
                  newcommand{fermi}{,{rm f}}
                  newcommand{floor}[1]{,leftlfloor #1 rightrfloor,}
                  newcommand{half}{{1 over 2}}
                  newcommand{ic}{{rm i}}
                  newcommand{iff}{Longleftrightarrow}
                  newcommand{imp}{Longrightarrow}
                  newcommand{pars}[1]{left(, #1 ,right)}
                  newcommand{partiald}[3]{frac{partial^{#1} #2}{partial #3^{#1}}}
                  newcommand{pp}{{cal P}}
                  newcommand{root}[2]{,sqrt[#1]{vphantom{large A},#2,},}
                  newcommand{sech}{,{rm sech}}
                  newcommand{sgn}{,{rm sgn}}
                  newcommand{totald}[3]{frac{{rm d}^{#1} #2}{{rm d} #3^{#1}}}
                  newcommand{verts}[1]{leftvert, #1 ,rightvert}$

                  $ds{sum_{n=1}^{infty} frac{1}{n^{3} binom{2n}{n}} = frac{ pi sqrt{3}}{18} Big[ psi_{1} left(frac{1}{3} right) - psi_{1} left(frac{2}{3} right) Big]- frac{4}{3} zeta(3)}$




                  begin{align}
                  &sum_{n = 1}^{infty}{x^{n} over n{2n choose n}}
                  =sum_{n = 1}^{infty}x^{n},
                  {Gammapars{n}Gammapars{n + 1} over Gammapars{2n + 1}}
                  =sum_{n = 1}^{infty}x^{n}int_{0}^{1}t^{n - 1}pars{1 - t}^{n},dd t
                  \[3mm] & =
                  int_{0}^{1}sum_{n = 1}^{infty}bracks{txpars{1 - t}}^{n}
                  ,{dd t over t}
                  =int_{0}^{1}bracks{{1 over 1 - txpars{1 - t}} - 1},{dd t over t}
                  \[3mm] & =
                  int_{0}^{1}{xpars{1 - t} over 1 - txpars{1 - t}},dd t
                  end{align}




                  begin{align}
                  &sum_{n = 1}^{infty}{x^{n} over n^{2}{2n choose n}}
                  =int_{0}^{x}dd yint_{0}^{1}{1 - t over 1 - typars{1 - t}},dd t
                  =-int_{0}^{1}{lnpars{1 - bracks{1 - t}tx} over t},dd t
                  end{align}




                  begin{align}
                  &color{#66f}{largesum_{n = 1}^{infty}{1 over n^{3}{2n choose n}}}
                  =-int_{0}^{1}{dd x over x}
                  int_{0}^{1}{lnpars{1 - bracks{1 - t}tx} over t},dd t
                  \[3mm] = &
                  int_{0}^{1}
                  {{rm Li}_{2}pars{tbracks{1 - t}} over t},dd t
                  \[3mm] = &
                  color{#66f}{{pi over 36root{3}}, left[psi ^{(1)}left(frac{1}{3}right)-psi ^{(1)}left(frac{2}{3}right)+psi ^{(1)}left(frac{1}{6}right)-psi ^{(1)}left(frac{5}{6}right)right]-frac{4 zeta (3)}{3}}
                  \[3mm] approx & {tt 0.5229}
                  end{align}







                  share|cite|improve this answer











                  $endgroup$



                  $newcommand{angles}[1]{leftlangle, #1 ,rightrangle}
                  newcommand{braces}[1]{leftlbrace, #1 ,rightrbrace}
                  newcommand{bracks}[1]{leftlbrack, #1 ,rightrbrack}
                  newcommand{ceil}[1]{,leftlceil, #1 ,rightrceil,}
                  newcommand{dd}{{rm d}}
                  newcommand{ds}[1]{displaystyle{#1}}
                  newcommand{expo}[1]{,{rm e}^{#1},}
                  newcommand{fermi}{,{rm f}}
                  newcommand{floor}[1]{,leftlfloor #1 rightrfloor,}
                  newcommand{half}{{1 over 2}}
                  newcommand{ic}{{rm i}}
                  newcommand{iff}{Longleftrightarrow}
                  newcommand{imp}{Longrightarrow}
                  newcommand{pars}[1]{left(, #1 ,right)}
                  newcommand{partiald}[3]{frac{partial^{#1} #2}{partial #3^{#1}}}
                  newcommand{pp}{{cal P}}
                  newcommand{root}[2]{,sqrt[#1]{vphantom{large A},#2,},}
                  newcommand{sech}{,{rm sech}}
                  newcommand{sgn}{,{rm sgn}}
                  newcommand{totald}[3]{frac{{rm d}^{#1} #2}{{rm d} #3^{#1}}}
                  newcommand{verts}[1]{leftvert, #1 ,rightvert}$

                  $ds{sum_{n=1}^{infty} frac{1}{n^{3} binom{2n}{n}} = frac{ pi sqrt{3}}{18} Big[ psi_{1} left(frac{1}{3} right) - psi_{1} left(frac{2}{3} right) Big]- frac{4}{3} zeta(3)}$




                  begin{align}
                  &sum_{n = 1}^{infty}{x^{n} over n{2n choose n}}
                  =sum_{n = 1}^{infty}x^{n},
                  {Gammapars{n}Gammapars{n + 1} over Gammapars{2n + 1}}
                  =sum_{n = 1}^{infty}x^{n}int_{0}^{1}t^{n - 1}pars{1 - t}^{n},dd t
                  \[3mm] & =
                  int_{0}^{1}sum_{n = 1}^{infty}bracks{txpars{1 - t}}^{n}
                  ,{dd t over t}
                  =int_{0}^{1}bracks{{1 over 1 - txpars{1 - t}} - 1},{dd t over t}
                  \[3mm] & =
                  int_{0}^{1}{xpars{1 - t} over 1 - txpars{1 - t}},dd t
                  end{align}




                  begin{align}
                  &sum_{n = 1}^{infty}{x^{n} over n^{2}{2n choose n}}
                  =int_{0}^{x}dd yint_{0}^{1}{1 - t over 1 - typars{1 - t}},dd t
                  =-int_{0}^{1}{lnpars{1 - bracks{1 - t}tx} over t},dd t
                  end{align}




                  begin{align}
                  &color{#66f}{largesum_{n = 1}^{infty}{1 over n^{3}{2n choose n}}}
                  =-int_{0}^{1}{dd x over x}
                  int_{0}^{1}{lnpars{1 - bracks{1 - t}tx} over t},dd t
                  \[3mm] = &
                  int_{0}^{1}
                  {{rm Li}_{2}pars{tbracks{1 - t}} over t},dd t
                  \[3mm] = &
                  color{#66f}{{pi over 36root{3}}, left[psi ^{(1)}left(frac{1}{3}right)-psi ^{(1)}left(frac{2}{3}right)+psi ^{(1)}left(frac{1}{6}right)-psi ^{(1)}left(frac{5}{6}right)right]-frac{4 zeta (3)}{3}}
                  \[3mm] approx & {tt 0.5229}
                  end{align}








                  share|cite|improve this answer














                  share|cite|improve this answer



                  share|cite|improve this answer








                  edited Dec 27 '18 at 16:55

























                  answered Sep 17 '14 at 4:17









                  Felix MarinFelix Marin

                  69.3k7110147




                  69.3k7110147






























                      draft saved

                      draft discarded




















































                      Thanks for contributing an answer to Mathematics Stack Exchange!


                      • Please be sure to answer the question. Provide details and share your research!

                      But avoid



                      • Asking for help, clarification, or responding to other answers.

                      • Making statements based on opinion; back them up with references or personal experience.


                      Use MathJax to format equations. MathJax reference.


                      To learn more, see our tips on writing great answers.




                      draft saved


                      draft discarded














                      StackExchange.ready(
                      function () {
                      StackExchange.openid.initPostLogin('.new-post-login', 'https%3a%2f%2fmath.stackexchange.com%2fquestions%2f499926%2fevaluating-the-sum-of-the-series-sum-limits-n-1-infty-frac1n3-bin%23new-answer', 'question_page');
                      }
                      );

                      Post as a guest















                      Required, but never shown





















































                      Required, but never shown














                      Required, but never shown












                      Required, but never shown







                      Required, but never shown

































                      Required, but never shown














                      Required, but never shown












                      Required, but never shown







                      Required, but never shown







                      Popular posts from this blog

                      Plaza Victoria

                      In PowerPoint, is there a keyboard shortcut for bulleted / numbered list?

                      How to put 3 figures in Latex with 2 figures side by side and 1 below these side by side images but in...